LSAT and Law School Admissions Forum

Get expert LSAT preparation and law school admissions advice from PowerScore Test Preparation.

User avatar
 Dave Killoran
PowerScore Staff
  • PowerScore Staff
  • Posts: 5853
  • Joined: Mar 25, 2011
|
#41357
Complete Question Explanation
(The complete setup for this game can be found here: lsat/viewtopic.php?t=2035)

The correct answer choice is (D)

Answer choice (A) is incorrect because G does not have two consecutive days off during the week (remember, the wording in the question stem is explicit: “at least two consecutive days off during the Monday-to-Friday workweek”). Answer choice (B) is incorrect because, according to the last rule, when H works then G must work on the following day. In this answer choice, H works on Monday, but G does not work on Tuesday. This answer choice also violates the third rule. Answer choice (C) is incorrect because I works on consecutive days, a violation of the third rule. Answer choice (E) is incorrect because F works later in the week than J, a violation of the fourth rule. Hence, answer choice (D) is correct.

Get the most out of your LSAT Prep Plus subscription.

Analyze and track your performance with our Testing and Analytics Package.